LSAT and Law School Admissions Forum

Get expert LSAT preparation and law school admissions advice from PowerScore Test Preparation.

 Administrator
PowerScore Staff
  • PowerScore Staff
  • Posts: 8919
  • Joined: Feb 02, 2011
|
#23628
Complete Question Explanation

Strengthen—PR. The correct answer choice is (D)

Here the author provides two examples in which participants were able to achieve the same results with less expensive and advanced equipment as those who had access to more cutting edge preparatory materials. The author spells out the conclusion in the final sentence: "one should not always purchase technologically advanced educational tools."

The question stem requires that we find the principle which strengthens (or, "most justifies") the author's conclusion.

Answer choice (A): The distinction described in this choice would play no role with regard to the author's conclusion.

Answer choice (B): Since the author's discussion surrounds how to make the personnel knowledgeable, this principle which tells us that such knowledge is important, has no effect on the author's argument.

Answer choice (C): The author doesn't believe that spending large amounts is necessarily justified in either context, so this answer choice cannot be the principle that we seek.

Answer choice (D): This is the correct answer choice. If this principle is valid, then it certainly strengthens the author's conclusion that one should not always purchase the most advanced educational tools.

Answer choice (E): The author does not argue for a variety of educational tools; the argument is about whether high-tech tools are justified when lower cost, lower tech tools do the job just as effectively.
 melissaantonia
  • Posts: 1
  • Joined: Nov 22, 2020
|
#81400
Hello, thank you for the explanation. However, I do not quite understand why this question is a Strengthen- PR, is it not a Justify question?
User avatar
 Dave Killoran
PowerScore Staff
  • PowerScore Staff
  • Posts: 5850
  • Joined: Mar 25, 2011
|
#81401
Hi Melissa,

Thanks for the question! It's close to a Justify question, but Justify requires 100% force. This stem includes the phrase "most helps" in it, which reduces that force. So the question drops from Justify down to Strengthen.

I hope that helps!

Get the most out of your LSAT Prep Plus subscription.

Analyze and track your performance with our Testing and Analytics Package.